PT 50 S2 Q14 help (LR) Forum

Prepare for the LSAT or discuss it with others in this forum.
Post Reply
mn40

New
Posts: 17
Joined: Thu Jan 23, 2014 11:49 pm

PT 50 S2 Q14 help (LR)

Post by mn40 » Sun Jul 13, 2014 8:23 pm

I'm not sure if it's appropriate to ask questions about specific PT questions on this forum. I apologize if that's the case.

I've really been struggling to figure out why D is the correct answer to PT 50 S2 Q14. I've read through the Manhattan forum replies but am still confused.

D seems like it would be a correct answer for a sufficient assumption question (e.g. "which of the following would allow the argument to be properly drawn?") rather than a necessary assumption question.

If I negate D and say: "legislators considering a proposed law for which they have repugnance or enthusiasm DO consider the consequences that it will actually have," it's still possible for legislatures to fail to enact laws that benefit constituents. Hence, the author does not depend on D as a necessary assumption.

Premise:
A --> B
Conclusion:
NOT A
Support:
repugnance/enthusiasm --> NOT A

To me, it seems like NOT A is still possible with B (instead of NOT B).

What am I missing? Any help would be greatly appreciated! Thank you in advance.

User avatar
WaltGrace83

Silver
Posts: 719
Joined: Fri Apr 12, 2013 5:55 pm

Re: PT 50 S2 Q14 help (LR)

Post by WaltGrace83 » Sun Jul 13, 2014 9:06 pm

(Enact laws that benefit constituents → must be sure to consider consequences)
+
Arouses repugnance or enthusiasm for legislation

~(Enact laws that benefit constituents)

The hard part about this question is understanding the core. Finding the true conclusion is super tricky and the true premises is also super tricky. However, we can use the THEREFORE test to understand how to do this in this particular question. It makes the most sense that the author would say that all of this information leads to the conclusion that "Contemporary legislations fail to enact laws that benefit the constituents."

Let me know if that doesn't make sense!

Now, how can we possibly - from a conditional premise in which we are given A → B, conclude ~A? There is one possible way to do this: We must assume ~B. If we have A → B, and we assume ~B, we can conclude ~A! This is just a simple contrapositive and I am assuming you know this.

Ok so the author is assuming that ~(must be sure to consider the consequences). That is, the author is assuming the negation of the necessary condition in order to conclude the negation of the sufficient condition given to us in the premises.

But like true LSAT fashion, our task is a bit more difficult than simply assuming that because we have another premise: (Arouses repugnance or enthusiasm for legislation). So the author is basically assuming that (Arouses repugnance or enthusiasm for legislation) is the equivalent to ~(must be sure to consider the consequences). That is, the author is assuming that this repugnance/enthusiasm stuff is synonymous with ~B. You got that?

  • There is also a more volatile, but perhaps efficient, way of solving the problem if you were really strapped for time on the real test. I can almost guarantee that, given the strange structure of this problem, there is going to be something to do with repugnance/enthusiasm. After all, it is our main premise that we are working off of. I would guess that the right answer would having something to do with that and (D) is the only one that does.

    (A) The "benefiting careers" bit is not really a premise but really a side-point or an explanation of why things are the way they are. It's not really a main topic of conversation.

    (B) 'successful" is just so off-the-mark its not even funny.

    (C) "adherence" is absolutely never talked about nor implied in any way.

    (E) We absolutely do not care WHY this is so.

User avatar
WaltGrace83

Silver
Posts: 719
Joined: Fri Apr 12, 2013 5:55 pm

Re: PT 50 S2 Q14 help (LR)

Post by WaltGrace83 » Sun Jul 13, 2014 9:08 pm

Also, a negated assumption doesn't mean that the conclusion is IMPOSSIBLE. It means that the CONCLUSION does not FOLLOW from the PREMISES.

Huge difference.

mn40

New
Posts: 17
Joined: Thu Jan 23, 2014 11:49 pm

Re: PT 50 S2 Q14 help (LR)

Post by mn40 » Mon Jul 14, 2014 12:32 am

That was very helpful - thank you!

User avatar
WaltGrace83

Silver
Posts: 719
Joined: Fri Apr 12, 2013 5:55 pm

Re: PT 50 S2 Q14 help (LR)

Post by WaltGrace83 » Mon Jul 14, 2014 10:05 am

No problem! I hope you get it now!

As I said, that is a super tough question because of the weird structure and how much stuff is going on at once. However, the answer choices really help us out here. :D

Want to continue reading?

Register now to search topics and post comments!

Absolutely FREE!


Post Reply

Return to “LSAT Prep and Discussion Forum”